<![CDATA[语时lab]]> 2020-04-26T14:53:22+08:00 https://cshishaliu.github.io/ MWeb <![CDATA[平面几何的全等和相似符号到底该怎么写]]> 2020-04-09T15:07:07+08:00 https://cshishaliu.github.io/15864160277643.html

这篇文章的原名本来计划是「\(\rm\LaTeX\) 中怎么输入平面几何的全等和相似符号」, 听上去满满的技术范, 但其实是一篇吐槽文章, 只为收集一点吐槽能量.

问题: 下图中有几种全等和相似符号的写法, 哪种才是对的呢?

我不知道你们看到这个问题的时候是什么感觉, 反正我发现这个居然会成为一个问题的时候, 心里感受到了一整个族群的神兽奔腾而过, 然后默念了一句「mdzz」。

背景简介

先解释一下这个问题为什么会成为问题, 原因主要有两点:

  • 真的会有 mdzz 老师因为全等或相似符号的“写法不对”而给学生扣分!!!1 而这些老师对此存有执念的原因, 大概主要是由于“官方” (人民教育出版社) 标准教材上, 非常严格且美观的使用了前面这个图中的某一个特定写法.
  • 目前普通人能用到的计算机排版工具以 Micro$oft Word 居多, 少数另类 (比方说我) 则会偏执的选择 \(\rm\TeX/\LaTeX\). 而似乎人民教育出版社并不使用这两种软件来制作出版物.

据说国内主要的出版社和杂志社报社使用的排版系统都是北大方正的产品, 我没见过这到底是一套什么样的排版系统, 只听闻是王选院士的作品.

然而很遗憾的是, 最近听到新闻说北大方正要被破产清算了, 不知道这个民族软件的优秀代表还能走多远, 是否有这样一种可能: 在可见的未来, 人民教育出版社也得另寻跟得上时代潮流的排版系统.

相关链接: [新浪财经] 北大方正破产:除了等待债券持有人还能做哪些事

我的辛路历程

由于最近写讲义的时候忽然意识到, 如果我写出来的全等符号跟人教版教材不一致, 我可能会被指为「不专业」. 于是, 我开始为这个事情进行了艰难的探索.

在目前的 Word 中, 至少有三种不同的方式来输入这些符号: 最基本的方式就是直接插入特殊符号, 另外就是借助 MathType 或自带的公式编辑器来输入. 我使用过后的感觉是, 这三种方法无论哪种在实际操作起来都谈不上简便的, 并且效果都有明显的瑕疵. 但考虑到用 Word 来排版技术/学术性文档, 尤其是那种带公式的, 本来就会有很多让人膈应但又只能强忍的瑕疵, 所以相比之下, 这三种方法排版出来的效果也都还行.

而在 \(\rm\LaTeX\) 中, 这个问题就要复杂不少, 当然其中一个主要原因是 \(\rm\LaTeX\) 在用起来本来就比 Word 要复杂不少.

在 \(\rm\LaTeX\) 中, 与这些符号相关的有三个命令, 具体名称和排版效果见下表.2

\(\rm\LaTeX\) 代码 效果
\cong \(\cong\)
\sim \(\sim\)
\backsim \(\backsim\)

可以看到, 这三个符号的排版效果跟人教版教材上的都有明显差别, 如果细抠的话, 连拐弯方向都是一个需要探讨的问题.

相关链接: [知乎] 中文字体和西文字体中全等号“≌”上半边为什么是反的?但相似号“∽”却是统一的,然而拉泰赫却只有“~”?

最终解决方案

MWE3

结论


  1. 我说/写出这句话的时候, 我也感受到了我对这种老师满满的恶意... 

  2. 这三个命令都是 amsmath 包提供的, 另有其它的包 (如 fdsymbols) 提供 \backcong 命令, 从名称上能感觉到 \backcong\cong 的关系应该跟 \backsim\sim 的关系类似. 

  3. MWE 的全称是 Minimal Workable Example, 貌似主要是 \(\rm\LaTeX\) 用户之间相互交流时发明的一个术语.  

]]>
<![CDATA[数列不等式 `lhc@jinan|20200407`]]> 2020-04-08T16:09:52+08:00 https://cshishaliu.github.io/15863333927200.html 已知数列 \(\{a_n\}\) 满足 \(a_1 = 1\), \(a_n a_{n+1} = n\), \(n = 1,2,3,\dots\). 求证:
\[
\frac1{a_1} + \frac1{a_2} + \frac1{a_3} + \dots + \frac1{a_n} \ge 2 \sqrt n - 1.
\]

Qer: lihaocheng@jinan 20200407

解答

这里给出一个解法, 过程相当简洁漂亮, 但有一个最大的问题就是, 似乎无法讲清楚这个思路是怎么来的, 这种方法是怎么想到的.

不难确认数列 \(\{a_n\}\) 的所有项都是正的, 由均值不等式
\[
\frac1{a_n} + \frac1{a_{n+1}} \ge 2 \sqrt{\frac1{a_n a_{n+1}}} = \frac2{\sqrt n}
\]
事实上, 这个等号也是取不到的, 不过这一点说起来比较费事, 不过后面可以看到也可以先不说这件事情.

于是, 当 \(n \ge 2\) 时,
\[
\begin{aligned}
&
\frac1{a_1} + \frac1{a_2} + \frac1{a_3} + \dots + \frac1{a_n} \\
{}={}&
\frac12 \left[
\frac1{a_1} + \left( \frac1{a_1} + \frac1{a_2} \right) + \left( \frac1{a_2} + \frac1{a_3} \right) + \dots + \left( \frac1{a_{n-1}} + \frac1{a_n} \right) + \frac1{a_n}
\right] \\
{}\ge{}&
\frac12 \left(
\frac1{a_1} + \frac2{\sqrt1} + \frac2{\sqrt2} + \dots + \frac2{\sqrt{n-1}} + \frac1{a_n}
\right) \\
{}={}&
\frac12 + \frac1{\sqrt1} + \frac1{\sqrt2} + \dots + \frac1{\sqrt{n-1}} + \frac1{2a_n}
\end{aligned}
\]

另一方面, 用数学归纳法不难证明
\[
\frac1{\sqrt1} + \frac1{\sqrt2} + \dots + \frac1{\sqrt{n-1}} \ge 2 \sqrt n - 1
\]

这样就不难证明原题中的不等式.

]]>
<![CDATA[如何判断复合根式是否可以进一步化简]]> 2020-04-08T11:30:03+08:00 https://cshishaliu.github.io/15863166036092.html

初学根式 (radical expression) 时, 经常遇到这样一类问题: 形如 \(\sqrt{A \pm \sqrt B}\) 的根式是否需要继续化简. 例如:

\(\sqrt{3 + 2\sqrt2}\) 可以化简为 \(\sqrt2 + 1\); \(\sqrt{2+\sqrt3}\) 可以化简为 \(\frac{\sqrt6 + \sqrt2}2\);

\(\sqrt{4 +\sqrt2}\) 似乎无法进一步化简.

这种根号里面还套着根号的式子其实在数学里面没有一个专门的术语名称, 本文中我们就把它叫做复合根式 (与之相对的, 没有根号嵌套的根式, 我们就称为简单根式).

在学生阶段做题的时候, 如果计算结果遇到这种复合根式, 通常是能进一步化简成不是复合形式的根式的. 但事实上, 如果我们能有更多的经验, 会发现「这种复合根式没法进一步化简」好像才是比较正常, 或者说, 比较常见、更容易发生的情况. 这个时候自然就有一个问题: 什么时候这种复合根式是可以继续化简的?

其实提到这个问题的时候, 我的内心还是有一点小骄傲的, 这也是我会写这篇文章的原因.

事情是这样的, 在很久很久以前我本人还是初中萌新的那个年代, 有一天数学老师在下课前最后不到一分钟的时间提到了这个问题, 并且以「掩耳不及迅雷之势」提点了一下答案, 但是由于这个过程实在太快, 几乎就没有同学注意到. 而作为极少数注意到这个知识点的同学, 我竟然还在接下来的课间 10 分钟内给出了完整的证明推导. 当时可把我牛 X 坏了, 我要先叉会儿腰去... :P

先说结论, 对于如下形式的符合根式
\[
\sqrt{A \pm \sqrt B}
\]
这里 \(A\) 和 \(B\) 都是正整数, 当且仅当 \(A^2 - B\) 是一个完全平方数时, 它可以化简成如下形式的简单根式
\[
\sqrt a \pm \sqrt b
\]
其中 \(a\) 和 \(b\) 要么是整数, 要么是分母为 \(2\) 的最简分数.

这个结论的证明其实也不难, 比方说我们就先考虑 \(\sqrt{A + \sqrt B}\); 稍微有计算经验的同学就应该能想到减号的情况 \(\sqrt{A - \sqrt B}\) 其实应该是类似的.

如果 \(A^2 - B\) 是一个完全平方数, 我们可以假设
\[
A^2 - B = n^2
\]
不难想到, 接下来的任务应该是设法把 \(A + \sqrt B\) 变形成一个完全平方式的样子; 并且主要变形技巧应该是利用上面刚刚设出的字母 \(n\) 来消掉原来的一个字母, 于是目前主要问题就是需要纠结一下到底消 \(A\) 还是消 \(B\).

如果你真的很纠结, 那我们就都试一试呗.

如果选择消 \(A\), 那就应该用到 \(A^2 = B + n^2 \Rightarrow A = \sqrt{B + n^2}\), 代入我们要研究的式子就有
\[
A + \sqrt B = \sqrt{B + n^2} + \sqrt B
\]
不出意外的话, 正常人看到这个式子应该是不会有任何想法的. 看上去此路不通, 那我们走另一条路看看: 要消 \(B\) 就应该用到 \(B = A^2 - n^2\), 于是
\[
A + \sqrt B = A + \sqrt{A^2 - n^2}
\]
这个式子乍看上去可能也会让人有点灰心, 但聪明如你应该至少能看出来可以用一次平方差公式,
\[
A + \sqrt{A^2 - n^2} = A + \sqrt{(A+n) (A-n)}
\]
然后接下来该怎么办就比较体现功底和毅力了. 你需要能想到以下两点, 并注意到它们之间的联系

  1. \(A+n\) 和 \(A-n\) 相加可以消去 \(n\) 得到只包含 \(A\) 的式子;
  2. 如果想让 \(A + \sqrt{(A+n) (A-n)}\) 套上完全平方公式的话, 其根式部分 \(\sqrt{(A+n) (A-n)}\) 似乎应可能是充当交叉项的角色.

然后在这两个点的指导下坚强的计算下去, 就能够成功的做出来.
\[
\begin{aligned}
A + \sqrt{(A+n) (A-n)}
&= \frac{(A+n) + (A-n)}2 + \sqrt{(A+n) (A-n)} \\
&= \frac{ \left( \sqrt{A+n} \right)^2 + \left( \sqrt{A-n} \right)^2 + 2 \cdot \sqrt{A+n} \cdot \sqrt{A-n}}2 \\
&= \frac{\left( \sqrt{A+n} + \sqrt{A-n} \right)^2}2
\end{aligned}
\]
这样就可以得到我们想要的结果
\[
\sqrt{A + \sqrt B} = \sqrt{\frac{\left( \sqrt{A+n} + \sqrt{A-n} \right)^2}2} = \sqrt{\frac{A+n}2} + \sqrt{\frac{A-n}2}
\]
类似的, 不难得到
\[
\sqrt{A - \sqrt B} = \sqrt{\frac{A+n}2} - \sqrt{\frac{A-n}2}
\]
至此, 我们的证明也基本上就完成了.

你还记得我们是想要证明什么吗?

不过, 问题到这里其实还可以有后续. 回去翻一下我们对待证结论的陈述方式

对于形如 \(\sqrt{A \pm \sqrt B}\) 的符合根式 (这里 \(A\) 和 \(B\) 都是正整数), 当且仅当 \(A^2 - B\) 是一个完全平方数时, 它可以化简成形如 \(\sqrt a \pm \sqrt b\) 的简单根式 (其中 \(a\) 和 \(b\) 要么是整数), 要么是分母为 \(2\) 的最简分数.

而目前我们得到的结论是
\[
\sqrt{A \pm \sqrt B} = \sqrt{\frac{A+n}2} \pm \sqrt{\frac{A-n}2}
\]
这里有 \(A^2 - B = n^2\). 我们至少还可以思考这么两个问题:

  1. 这个「公式」的左边是 \(A,B\) 两个字母, 但右边却「凭空」出现了一个 \(n\), 能不能把式子的形式写的更「合理」一点呢?
  2. 注意一下我们前面对待证结论最早的陈述方式, 措辞中用到了一个词: 「当且仅当」. 这里面其实还有一个反问题, 即

    如果形如 \(\sqrt{A \pm \sqrt B}\) 的符合根式可以化简成形如 \(\sqrt a \pm \sqrt b\) 的简单根式, 那么 \(A^2 - B\) 是否一定是一个完全平方数?

各位读者同学, 这两个问题你们都有思路吗?

]]>
<![CDATA[Simon Tatham's Portable Puzzle Collection]]> 2020-04-04T16:32:08+08:00 https://cshishaliu.github.io/15859891289445.html

Simon Tatham's Portable Puzzle Collection (以下简称 SGT Puzzles), 顾名思义是一个谜题的合集 (Puzzle Collection), Simon Tatham 是它的作者, 而 Portalble 在这里我认为有两重意思:

  1. 这些谜题游戏都很小巧, 它们都是一个人玩的, 并且一局的时间通常只有几分钟;
  2. 这个谜题合集作为一个软件包, 可以轻易的运行在多种不同的操作系统上, 即计算机专业说的 portable (可移植).

SGT Puzzles 中包含的谜题

SGT Puzzles 中目前 (2020 年 4 月) 包含 39 个不同谜题, 包括:

  • black box,
  • bridges,
  • cube,
  • dominosa,
  • fifteen,
  • filling,
  • flip,
  • flood,
  • galaxies,
  • guess,
  • inertia,
  • keen,
  • light up,
  • loopy,
  • magnets,
  • map,
  • mines,
  • net,
  • netslide,
  • palisade,
  • pattern,
  • pearl,
  • pegs,
  • range,
  • rect,
  • same game,
  • signpost,
  • singles,
  • sixteen,
  • slant,
  • solo,
  • tents,
  • towers,
  • tracks,
  • twiddle,
  • undead,
  • unequal,
  • unruly,
  • untangle.



其中甚至有我们非常常见的数独和扫雷. 更有意思的是, SGT Puzzles 中的扫雷还允许撤销, 也就是说, 按照一般扫雷游戏的规则, 你永远都能赢. 但是在 SGT Puzzles 的扫雷中, 会记录你触雷“死亡”的次数. 如果你很在意这个次数, 那么你会发现撤销功能并不能减轻你扫雷时的心理压力.

有不少谜题在别的 App 中也能看到, 但通常一个 App 就是一个谜题, 而在这里你能看到最全的.

关于作者 Simon Tatham

Simon Tatham 是英国 ARM 公司的一名软件工程师, 通俗的说也可以算是一个程序员, 或者叫做「码农」.

Simon Tatham 是世界顶级的程序员, 有很多很棒的软件作品, 知名度甚至比他本人还要高, 如 PuTTY. SGT Puzzles 中的大多数谜题都不是 Simon Tatham 发明的, 他只是搜集了很多各种各样的谜题游戏 (包括他发明的), 为它们写了一个共同的程序框架, 然后再将它们一一实现. 根据他 (或者说 SGT Puzzles) 的主页上的描述, 他写 SGT Puzzles 只是因为希望在电脑桌面上放一些随时可玩的小游戏, 并且希望不受到操作系统平台的限制.

I wrote this collection because I thought there should be more small desktop toys available: little games you can pop up in a window and play for two or three minutes while you take a break from whatever else you were doing.
And I was also annoyed that every time I found a good game on (say) Unix, it wasn't available the next time I was sitting at a Windows machine, or vice versa; so I arranged that everything in my personal puzzle collection will happily run on both those platforms and more.

如何下载/获取 SGT Puzzles

官方网站

SGT Puzzles 的主页地址为 https://www.chiark.greenend.org.uk/~sgtatham/puzzles/. 在这个页面上提供了:

  • JavaScriptJava Applet 实现的谜题游戏, 可以在网页上直接体验.
  • .exe 格式的 Windows 可执行文件, 以及 .dmg 格式的 Mac OS X 安装包, 均可直接下载.
  • 所有谜题的完整说明文档, 有 html 页面的链接, 也提供 chm 格式文件下载, 但只有英文版的, 且到目前 (2020 年 4 月) 为止, 似乎没有人做过汉化工作.
  • 其它移动平台的安装方式参考链接, 包括: Android 和 iOS (iPhone/iPad).
  • 另外还有所有源程序代码包, 以及 git 代码仓库的地址.

目前, 在苹果应用商店 App Store 里的商品链接为

https://apps.apple.com/cn/app/simon-tathams-portable-puzzle-collection/id622220631

在微软应用商店里的商品链接为

https://www.microsoft.com/zh-cn/p/portable-puzzle-collection/9nblggh16n44?rtc=1&activetab=pivot:overviewtab

]]>
<![CDATA[Books]]> 2020-04-01T15:18:10+08:00 https://cshishaliu.github.io/book.html 这个页面中是我推荐的书籍.

思考的乐趣, 顾森

豆瓣评分: 8.2.

]]>
<![CDATA[Links]]> 2020-04-01T11:50:30+08:00 https://cshishaliu.github.io/links.html 这个页面中是一些我比较喜欢的网站.

matrix67 blog

名噪一时的 matrix67 数学博客: http://www.matrix67.com/blog/

matrix671北大中文系本科生, 相传是信息学竞赛保送去的. 一位信息竞赛选手去了中文系是一件让人很费解的事情, 有一个合理的解释: 是当年的北大中文系和计算机系合办了一个交叉学科专业, 叫做应用语言学, 然后就把他忽悠去了.

matrix67 数学博客貌似早在作者本人高中阶段就办起来了, 持续更新了十多年. 截止目前2, 最后一篇更新文章「Lissajous 曲线的动画演示」的发布时间为 2016-10-18, 且其短无比, 摘录如下.

随着常数 m 和 n 的变化,参数方程 x = sin(m · t), y = sin(n · t) 将会画出一系列漂亮的曲线。法国物理学家 Jules Antoine Lissajous 曾在 1857 年研究过这类曲线,因此人们把它叫做 Lissajous 曲线。我在 reddit 上看到了一个 Lissajous 曲线的动画演示,觉得看起来确实非常爽;但那个动画里没有解释曲线的生成方法,很多细节也有让人不太满意的地方,于是决定自己制作一个。这个动画展示的是 m = 13, n = 18 时的 Lissajous 曲线。

但总的来说, matrix67 数学博客上的文章质量大都很高, 也基本没有特别高深的数学理论, 特别适合基础一般且对数学有特殊兴趣的中学生和业余数学爱好者.

在 matrix67 小有名气以后, 人民邮电出版社联系他出版了一本书「思考的乐趣」, 这本书写得也很棒, 目前3豆瓣评分 8.2.

Art of Problem Solving

大名鼎鼎的 Art of Problem Solving: https://artofproblemsolving.com. 这个网站在数竞圈里应该是无人不知无人不晓了.


  1. 相传 matrix 67 给自己取了这么一个 ID 的原因, 不是因为他有多么喜欢线性代数或者矩阵理论, 而是喜欢一部名叫「matrix」的电影. 于是在给自己的 blog 注册域名的时候, 他尝试了 matrix, matrix123 和 matrix45, 发现这些域名都已经被注册了, 一直到 matrix67 才是空闲的. 

  2. 这里的「目前」, 日期是 2020-04-01. 

  3. 这里的「目前」, 日期是 2020-04-01. 

]]>
<![CDATA[二元有理函数最值]]> 2020-03-13T08:45:46+08:00 https://cshishaliu.github.io/15840603466913.html 已知 \(x,y > 0\), 求
\[
\frac{(2x+1) (y+1)}{2x^2 + 5y^2 + 7}
\]
的最大值.

Qer: QQ 群: 中国数学竞赛交流群 20200317

解答

设 \(t = \frac{2x^2 + 5y^2 + 7}{(2x+1) (y+1)}\), 显然 \(t>0\), 且去分母并移项有
\[
2x^2 - 2txy + 5y^2 - 2t x - t y - t + 7 = 0
\]
此式可变形为
\[
2x^2 - 2t (y+1) x + [5y^2 + 7 - t(y+1)] = 0
\]
从而视作关于 \(x\) 的一元二次方程, 于是应有
\[
\Delta_1 = 4t^2(y+1)^2 - 8 [5y^2 + 7 - t(y+1)] \ge 0
\]
整理得
\[
(t^2 - 10) y^2+ (2t^2 +2t) y + (t^2 + 2t - 14) \ge 0
\]
将它视作关于 \(y\) 的一元二次不等式, 则当 \(t < \sqrt{10}\) 时, 由于二次项系数 \(t^2 - 10 \le 0\), 应有
\[
\Delta_2 = (2t^2 + 2t)^2 - 4(t^2-10)(t^2+2t-14) \ge 0
\]
化简可得
\[
5t^2 + 4t - 28 \ge 0
\]
解得
\[
t \le -\frac{14}5 ~\text{或}~ t \ge 2
\]
因此, 当 \(t < \sqrt{10}\) 时, \(t\) 的最小正值为 \(2\), 此时由
\[
\begin{aligned}\Delta_1 &= 4t^2(y+1)^2 - 8 [5y^2 + 7 - t(y+1)] \\&= 16 (y+1)^2 - 8 [5y^2 + 7 - 2(y+1)] \ge 0\end{aligned}
\]
可解得 \(y=1\), 进一步由 \(t=2,y=1\) 可计算得到 \(x=2\).

综上所述, 在 \(t<\sqrt{10}\) 的前提条件下, \(t\) 在 \(x=2,y=1\) 时取得最小值 \(2\). 于是 \(t \ge \sqrt{10}\) 的情况就不用再讨论了.

快速解法

由于
\[
\begin{aligned}
2x^2 + 5y^2 + 7
&= (x^2 + 4y^2) + (x^2 + 4) + (y^2 + 1) + 2 \\
&\ge 4xy + 4x + 2y + 2 \\
&= 2(2x+1) (y+1)
\end{aligned}
\]
当且仅当 \(x = 2y = 2\) 时取等号, 因此
\[
\frac{(2x+1) (y+1)}{2x^2 + 5y^2 + 7} \le \frac12
\]

]]>
<![CDATA[两个数列不等式问题]]> 2019-02-01T15:57:59+08:00 https://cshishaliu.github.io/15490078791835.html
  • 已知实数 \(x_1, x_2, \dots, x_{10}​\), \(\sum\limits_{k=1}^{10} k x_k = 1​\). 求 \(\left( \sum\limits_{k=1}^{10} x_k \right)^2 + \sum\limits_{k=1}^{10} x_k^2​\) 的最小值.
  • 设 \(a_n = 1 + \dfrac12 + \dfrac13 + \dots + \dfrac1n\). 求证: 对 \(n\ge 2\) 有 \(a_n^2 < 2 \left( \dfrac{a_2}2 + \dfrac{a_3}3 + \dots \dfrac{a_n}n \right) + \dfrac{n+3}{2n+2}\).
  • Qer: xieshunfu 20190201

    解答

    题 1

    对于任意实数 \(t\), 均有 (Cauchy 不等式):
    \[
    \begin{aligned}
    \left[ \left( \sum_{k=1}^{10} x_k \right)^2 + \sum_{k=1}^{10} x_k^2 \right]
    \left[ t^2 + \sum_{k=1}^{10} (k-t)^2 \right]
    &\ge
    \left[ \left( \sum_{k=1}^{10} x_k \right) \cdot t + \sum_{k=1}^{10} (k-t) x_k \right]^2 \\
    &= \left( \sum_{k=1}^{10} k x_k \right)^2 = 1
    \end{aligned}
    \]

    \[
    \left[ \left( \sum_{k=1}^{10} x_k \right)^2 + \sum_{k=1}^{10} x_k^2 \right]
    \cdot (11 t^2 - 110 t + 385) \ge 1
    \]

    \[
    \left( \sum_{k=1}^{10} x_k \right)^2 + \sum_{k=1}^{10} x_k^2
    \ge \dfrac1{11 t^2 - 110 t + 385}
    \]
    此不等式对一切 \(t \in \mathbb R\) 都成立. 因此
    \[
    \left( \sum_{k=1}^{10} x_k \right)^2 + \sum_{k=1}^{10} x_k^2
    \ge \left( \dfrac1{11 t^2 - 110 t + 385} \right)_{\max} =
    \left( \dfrac1{11 t^2 - 110 t + 385} \right)_{t=5} = \dfrac1{110}
    \]
    当且仅当
    \[
    \begin{aligned}
    \left( \sum_{k=1}^{10} x_k \right) : x_1 : x_2 : \dots : x_{10}
    &= t : (1-t) : (2-t) : \dots : (10-t) \\
    &= 5 : (-4) : (-3) : \dots : 5
    \end{aligned}
    \]
    时取等号.

    题 2

    数学归纳法, 不难!

    ]]>
    <![CDATA[CMO 2018 题 3]]> 2018-11-14T20:20:30+08:00 https://cshishaliu.github.io/15421980305249.html TODO: 微博图床已挂, 本篇缺图, 需要找回

    \(\triangle ABC\) 中, \(AB < AC\), \(O\) 为外心, \(D\) 是 \(\angle BAC\) 平分线上一点, \(E\) 在 \(BC\) 上, 满足 \(OE \parallel AD\), \(DE \perp BC\). 在射线 \(EB\) 上取点 \(K\) 满足 \(EK = EA\), \(\triangle ADK\) 外接圆与 \(BC\) 交于另一点 \(P \ne K\), \(\triangle ADK\) 外接圆与 \(\triangle ABC\) 外接圆交于另一点 \(Q \ne A\). 求证: \(PQ\) 与 \(\triangle ABC\) 外接圆相切.

    题目来源: 2018冬令营首日学生回忆版试题 (爱尖子发布)

    解答

    设 \(\triangle ABC\) 的外心为 \(O\), \(\triangle ADK\) 的外心为 \(O_1\). 连结 \(AO\), \(DO\), \(AO_1\), \(KO_1\) 和 \(EO_1\). 设 \(OD\) 与 \(BC\) 的交点为 \(F\).

    不难证明 \(\angle OAD = \angle ADE = \dfrac{B-C}2\), 故 \(ADEO\) 是等腰梯形.

    于是由 \(O_1A = O_1D\) 知 \(O_1O = O_1E\), 且 \(OD = AE = EK\).

    因此, \(\triangle O_1KE \cong \triangle D_1DO\), 故 \(\angle KO_1D = \angle KFD = \angle EO_1O\).

    因为 \(EO_1\) 垂直平分 \(AK\), 且 \(O_1O\) 垂直平分 \(AQ\), 所以 \(\angle EO_1O = \angle KAQ = 180^\circ - \angle KPQ\).

    于是 \(\angle KFO = 180^\circ - \angle KFD = 180^\circ - \angle EO_1O = \angle KPQ\), 故有 \(OD \parallel PQ\).

    另外, 由 \(\angle KFD = \angle KO_1D = 2 \angle KPD\) 知 \(\angle FPD = \angle FDP\), \(FP = FD\).

    因此, 点 \(P\) 到 \(FD\) 的距离与点 \(D\) 到 \(FP\) 的距离相同, 都等于 \(DE = OA\).

    即, 平行线 \(OD\) 与 \(PQ\) 之间的距离等于 \(\triangle ABC\) 外接圆 (圆 \(O\)) 的半径, 因此 \(PQ\) 与圆 \(O\) 相切.

    ]]>
    <![CDATA[两个二次函数问题]]> 2018-10-15T15:15:04+08:00 https://cshishaliu.github.io/15395877048195.html
  • 已知二次函数 \(f(x) = a x^2 + b x + c\) 的图象过点 \((2,8)\), 且对一切实数 \(x\) 恒有 \(2x + 3 \le f(x) \le 2x^2 - 2x + 5\), 求 \(f(x)\).
  • 在平面直角坐标系中, 不论 \(m\) 取何值时, 抛物线 \(y = mx^2 + (2m+1) x - (3m+2)\) 都不通过的直线 \(y = -x+1\) 上的点有哪些? (写出全部符合条件点的坐标)
  • Qer: XUC 20181014

    题 1

    依题意, \(f(2) = 4a + 2b + c = 8\). 另外, 在已知条件中令 \(x = 1\), 有 \(5 \le f(1) \le 5\), 因此, \(f(1) = a+b+c = 5\). 这两组等式可解得 \(\begin{cases} b = 3-3a \\ c = 2+2a \end{cases}\), 于是 \(f(x) = ax^2 + (3-3a)x + (2+2a)\).

    又依题意 \(f(x) - (2x+3) = a x^2 + (1-3a) x + (-1+2a) \ge 0\) 恒成立, 于是 \(a > 0\) 且
    \[
    \Delta = (1-3a)^2 - 4a(-1+2a) = a^2 - 2a + 1 \le 0
    \]
    因此, \(a = 1\). 故 \(b=0\), \(c=4\), \(f(x) = x^2 +4\).

    另外, 又 \(f(x) - (2x^2 - 2x + 5) = (a-2) x^2 + (5-3a) x + (-3+2a) \le 0\) 恒成立, 有 \(a-2 < 0\) 且
    \[
    \Delta = (5-3a)^2 - 4(a-2)(-3+2a) = a^2 - 2a +1 \le 0
    \]
    解得的也是 \(a = 1\).

    题 2

    联立抛物线和直线方程, 变形可得
    \[
    m (x^2 + 2x - 3) = -2x + 3
    \]
    将它视作关于 \(m\) 的一元一次方程, 则 \(x = 1\) 或 \(-3\) 时, 此方程无解. 这个结果可以解释为:
    无论 \(m\) 取何值, 抛物线和直线的交点横坐标都不可能是 \(1\) 或 \(-3\); 而对于其它可能的交点横坐标值, 都有合适的 \(m\) 取值与之对应.

    直线 \(y = -x+1\) 上, 横坐标 \(1\) 和 \(-3\) 所对应的纵坐标分别是 \(0\) 和 \(4\), 因此抛物线不会通过的直线上的点只有 \((1,0)\) 和 \((-3,4)\) 两个.

    ]]>
    <![CDATA[构造高次多项式解决多元方程问题]]> 2018-08-29T15:54:44+08:00 https://cshishaliu.github.io/15355292847712.html 已知 \(x_1, x_2, \dots, x_9\) 满足方程组
    \[
    \begin{cases}
    \dfrac{x_1}{11} + \dfrac{x_2}{12} + \dots + \dfrac{x_9}{19} = 1 \\
    \dfrac{x_1}{21} + \dfrac{x_2}{22} + \dots + \dfrac{x_9}{29} = 1 \\
    \dfrac{x_1}{31} + \dfrac{x_2}{32} + \dots + \dfrac{x_9}{39} = 1 \\
    \dfrac{x_1}{41} + \dfrac{x_2}{42} + \dots + \dfrac{x_9}{49} = 1 \\
    \dfrac{x_1}{51} + \dfrac{x_2}{52} + \dots + \dfrac{x_9}{59} = 1 \\
    \dfrac{x_1}{61} + \dfrac{x_2}{62} + \dots + \dfrac{x_9}{69} = 1 \\
    \dfrac{x_1}{71} + \dfrac{x_2}{72} + \dots + \dfrac{x_9}{79} = 1 \\
    \dfrac{x_1}{81} + \dfrac{x_2}{82} + \dots + \dfrac{x_9}{89} = 1 \\
    \dfrac{x_1}{91} + \dfrac{x_2}{92} + \dots + \dfrac{x_9}{99} = 1 \\
    \end{cases}
    \]
    试求 \(x_1 + x_2 + \dots + x_9\).

    解答

    构造 (关于 \(n\) 的) 多项式
    \[
    f(n) = \left[1 - \left(\dfrac{x_1}{n+1} + \dfrac{x_2}{n+2} + \dots + \dfrac{x_9}{n+9}\right)\right](n+1)(n+2)\cdots(n+9)
    \]

    \[
    f(10) = f(20) = \dots = f(90) = 0
    \]
    这样 \(9\) 次多项式 \(f(n)\) 根全部确定, 因此可以假设
    \[
    f(n) = a(n-10) (n-20) \dots (n-90)
    \]
    对比最高次项 (\(9\) 次项) 的系数可得 \(a = 1\), 即
    \[
    \left[1 - \left(\dfrac{x_1}{n+1} + \dfrac{x_2}{n+2} + \dots + \dfrac{x_9}{n+9}\right)\right](n+1)(n+2)\cdots(n+9) = (n-10) (n-20) \dots (n-90)
    \]
    又对比 \(8\) 次项的系数可得
    \[
    (1 + 2 + \dots + 9) - (x_1 + x_2+ \dots + x_9) = (-10) + (-20) + \dots + (-90)
    \]
    所以 \(x_1 + x_2 + \dots + x_9 = 495\).

    ]]>
    <![CDATA[伪装成函数方程的数论问题]]> 2018-08-22T09:06:12+08:00 https://cshishaliu.github.io/15348999728702.html 已知函数 \(f(x,y)\) 定义在正整数集上, 满足 \(\forall x,y \in \mathbb N^*\),

    1. \(f(x,x) = x\),
    2. \(f(x,y) = f(y,x)\),
    3. \((x+y) \cdot f(x,y) = y \cdot f(x, x+y)\).

    求证: \(f(x,y) = [x,y]\) (这里, \([x,y]\) 表示 \(x\) 与 \(y\) 的最小公倍数).

    Qer: zhangboxin 20180821

    探究

    这道题出成证明题已经大大降低了难度, 如果出成解答题, 要求 \(f(x,y)\) 的解析式, 就会难很多.

    这是一个数论和函数方程的问题, 作为函数方程的问题, 标准的上手思路当然还是代入一些特殊值来寻找规律并验证结论. 这里代值的方法当然主要是针对已知条件 3, 令 \(y = x\) 有
    \[
    2x \cdot f(x,x) = x \cdot f(x, 2x) \Rightarrow f(x, 2x) = 2 f(x,x) = 2x
    \]
    又令 \(y = 2x\), 有
    \[
    3x \cdot f(x, 2x) = 2x \cdot f(x, 3x) \Rightarrow f(x, 3x) = \dfrac32 f(x,2x) = 3x
    \]
    以此类推, 不难证明
    \[
    \forall n, x \in \mathbb N^*, f(x, nx) = nx
    \]
    再结合条件 2 知, 当 \(x\) 和 \(y\) 有倍数关系 (即 \(y=nx\) 或 \(x=ny\)) 的时候, 结论 \(f(x,y) = [x,y]\) 是成立的. 那么, 它们没有倍数关系的时候呢? 这时可以考虑利用条件 3 来对 \(f(x,y)\) 做变形和转换, 设法使得两个数变得有倍数关系.

    在条件 3 中用 \(x+y\) 替换 \(y\) 有
    \[
    (2x+y) \cdot f(x,x+y) = (x+y) \cdot f(x, 2x+y)
    \]
    两边同时乘以 \(y\), 再结合条件 3 本身, 有
    \[
    (x+y) \cdot y \cdot f(x, 2x+y) = (2x+y) \cdot y \cdot f(x,x+y) = (2x+y) (x+y) f(x,y)
    \]
    由于 \(x+y\) 是正整数, 大于零, 因此
    \[
    (2x+y) \cdot f(x,y) = y \cdot f(x, 2x+y)
    \]
    类似的, 依次用 \(2x+y, 3x+y, \dots\) 替换 \(y\) 可以得到
    \[
    \forall n \in \mathbb N^*, (nx + y) \cdot f(x,y) = y \cdot f(x, nx+y)
    \]
    事实上, 这个式子不仅对于正整数 \(n\) 成立, 在 \(y > x\) 时对于负的整数 \(n\) 也是成立的, 只要确保 \(nx + y > 0\) 这个前提. 这就让人联想到可以借助辗转相除法.

    另外, 如果注意到条件 3 形式上的对称性, 将其乘式形式变成除式形式, 并做适当变形, 可以得到一个更容易处理的形式:
    \[
    \dfrac{f(x,y)}{x \cdot y} = \dfrac{f(x, x+y)}{x \cdot (x+y)}
    \]
    显然, 需要把 \(\dfrac{f(x,y)}{x \cdot y}\) 视作一个整体, 那么这个等式其实也在提醒我们使用辗转相除法.

    解答

    构造函数 \(g(x,y) = \dfrac{f(x,y)}{x \cdot y}\), 则

    1. \(g(x,x) = \dfrac1x\);
    2. \(g(x,y) = g(y,x)\);
    3. \(g(x,y) = g(x, x+y)\).

    为证明 \(f(x,y) = [x,y]\), 只需证明 \(g(x,y) = \dfrac1{(x,y)}\), 这里 \((x,y)\) 表示 \(x\) 与 \(y\) 的最大公约数.

    对于任意 \(x,y \in \mathbb N^*\), 若 \(x = y\), 则结论显然成立. 否则, 不妨设 \(x < y\), 则可作带余除法 \(y = x \cdot q_1 + r_1\) (其中 \(q_1, r_1 \in \mathbb N^*\) 且 \(r_1 < x\)), 于是由等式 3 有
    \[
    g(x, r_1) = g(x, x+r_1) = g(x, 2x+r_1) = \dots = g(x, q_1 \cdot x + r_1) = g(x, y)
    \]
    若 \(r_1 | x\), 则
    \[
    g(r_1, r_1) = g(r_1, 2r_1) = g(r_1, 3r_1) = \dots = g(r_1, x)
    \]
    因此
    \[
    g(x,y) = g(x, r_1) = g(r_1, x) = g(r_1, r_1) = \dfrac1{r_1}
    \]
    而不难证明此时 \(r_1 = (x,y)\), 因而结论成立.

    否则 (若 \(r_1 \nmid x\)) 可继续作带余除法 \(x = r_1 \cdot q_2 + r_2\) (其中 \(q_2, r_2 \in \mathbb N^*\) 且 \(r_2 < r_1\)), 于是
    \[
    g(r_1, r_2) = g(r_1, r_1 + r_2) =
    g(r_1, 2r_1 + x_2) = \dots = g(r_1, q_2 \cdot r_1 + r_2) = g(r_1, x)
    \]
    而 \(g(x,y) = g(x, r_1) = g(r_1, x) = g(r_1, r_2) = g(r_2, r_1)\). 若 \(r_2 | r_1\), 则不难证明这个值是 \(\dfrac1{r_2}\), 且此时 \(r_2 = (x,y)\). 否则 (若 \(r_2 \nmid r_1\)) 继续作带余除法 \(r_1 = r_2 \cdot q_3 + r_3\)...

    以此类推, 这样继续下去可以得到一个有限项的恒为正值且单调递减的余数数列 \(r_1 > r_2 > \dots > r_{n-1} > r_n\), 满足 \(r_{k-2} = r_{k-1} \cdot q_k + r_k\) (\(k = 3,4,\dots,n\)), 这个数列在第 \(n\) 项终止当且仅当 \(r_n | r_{n-1}\). 不难证明
    \[
    g(x, y) = g(x, r_1) = g(r_1, r_2) = g(r_2, r_3) = \dots = g(r_{n-1}, r_n) = \dfrac1{r_n}
    \]

    \[
    (x, y) = (x, r_1) = (r_1, r_2) = (r_2, r_3) = \dots = (r_{n-1}, r_n) = r_n
    \]
    因而结论成立.

    ]]>
    <![CDATA[二次根式化简]]> 2018-08-08T13:53:38+08:00 https://cshishaliu.github.io/15337076183356.html 计算:
    \[
    \dfrac{
    1 + \sqrt{2-\sqrt2} + \sqrt{2-\sqrt3}
    }{
    \sqrt3 + \sqrt{2+\sqrt2} + \sqrt{2+\sqrt3}
    }
    \]

    Qer: zhaozhe 20180807

    解答

    \[
    \begin{aligned}
    \text{原式}
    &= \dfrac{
    1 + \sqrt{2-\sqrt2} + \dfrac{\sqrt6-\sqrt2}2
    }{
    \sqrt3 + \sqrt{2+\sqrt2} + \dfrac{\sqrt6+\sqrt2}2
    } \\
    &= \dfrac{
    2\sqrt{2-\sqrt2} + 2 + \sqrt6 - \sqrt2
    }{
    2\sqrt{2+\sqrt2} + 2\sqrt3 + \sqrt6 + \sqrt2
    }
    \end{aligned}
    \]
    不难验证,
    \[
    (2\sqrt3 + \sqrt6 + \sqrt2) (\sqrt2 - 1) = 2 + \sqrt6 - \sqrt2
    \]
    以及
    \[
    \dfrac{\sqrt{2-\sqrt2}}{\sqrt{2+\sqrt2}} =
    \sqrt{\dfrac{2-\sqrt2}{2+\sqrt2}} = \sqrt2 - 1
    \]
    因此, 原式 \(=\sqrt2-1\).

    ]]>
    <![CDATA[liujihang 20180801]]> 2018-08-01T22:23:11+08:00 https://cshishaliu.github.io/15331333919370.html TODO: 微博图床已挂, 本篇缺图, 需要找回

    无穷数列 \(P \colon a_1, a_2, \dots, a_n, \dots\) 满足 \(a_i \in \mathbb N^*\), 且 \(a_i \le a_{i+1}\) (\(i\in \mathbb N^*\)). 对于数列 \(P\), 记 \(T_k (P) = \min \{ n | a_n \ge k \}\) (\(k\in \mathbb N^*\)), 其中 \(\min \{ n | a_n \ge k \}\) 表示集合 \(\{ n | a_n \ge k \}\) 中最小的数.

    1. 若数列 \(P\colon 1,3,4,7,\dots\), 写出 \(T_1(P), T_2(P), \dots, T_5(P)\);
    2. 若 \(T_k (P) = 2k-1\), 求数列 \(P\) 的前 \(n\) 项之和;
    3. 已知 \(a_{20} = 46\), 求 \(s = a_1 + a_2 + \dots + a_{20} + T_1 (P) + T_2 (P) + \dots + T_{46} (P)\) 的值.

    解答

    1. \(T_1(P) = 1\), \(T_2(P) = 2\), \(T_3(P)=2\), \(T_4(P)=3\), \(T_5(P) = 4\).

    2. 不难得出满足 \(T_k (P) = 2k-1\) 的数列为 \(P\colon 1,1,2,2,3,3,4,4,\dots\), 其通项公式可以写作 \(a_n = \dfrac{2n+1-(-1)^n}4\). 其前 \(n\) 项的和为 \(S_n = \dfrac{2n^2 + 4n + 1 - (-1)^n}8\).

    3. 我们证明一个更一般的结论: 记
      \[
      s_n = a_1 + a_2 + \dots + a_n + T_1(P) + T_2(P) + \dots + T_{a_n}(P)
      \]

      \[
      s_n = (n+1) a_n
      \]
      采用数学归纳法证明.

      1. \(n=1\) 时,
        \[
        s_1 = a_1 + T_1(P) + T_2(P) + \dots + T_{a_1}(P) = a_1 + \underbrace{1+1+\dots+1}_{a_1~\text{个}~1} = 2 a_1
        \]
        结论成立.

      2. 假设 \(n = m-1\) 时结论成立, 即有
        \[
        s_{m-1} = a_1 + a_2 + \dots + a_{m-1} + T_1(P) + T_2(P) + \dots + T_{a_{m-1}}(P) = m a_{m-1}
        \]
        则 \(n = m\) 时, 若 \(a_m = a_{m-1}\), 则 \(s_m\) 相比于 \(s_{m-1}\) 中只增加了一项 \(a_m\) (注意此时 \(T_{a_m}(P)\) 就是 \(T_{a_{m-1}}(P)\)), 因此
        \[
        s_m = s_{m-1} + a_m = m a_{m-1} + a_m = (m+1) a_m
        \]
        若 \(a_m > a_{m-1}\), 则 \(s_m\) 相比于 \(s_{m-1}\) 中增加的项有: \(a_m\), \(T_{a_{m-1}+1}(P), T_{a_{m-1}+2}(P), \dots, T_{a_m}(P)\), 共有 \(a_m - a_{m-1} + 1\) 项. 其中, \(T_{a_{m-1}+1}(P) = T_{a_{m-1}+2}(P) = \dots = T_{a_m}(P) = m\), 因此
        \[
        s_m = s_{m-1} + a_m + (a_m - a_{m-1}) m = (m+1) a_m
        \]
        证毕.

      因此, 原题中要求的和数为 \(s_{20} = (20+1) \times 46 = 966\).

    附图: 本题的几何意义

    ]]>
    <![CDATA[不动点为复数值的递推数列问题]]> 2018-07-28T21:58:47+08:00 https://cshishaliu.github.io/15327863271008.html 正数数列 \(\{a_n\}\) 满足 \(S_n + 1 = \dfrac{S_n+4}{2a_n - S_n}\), 其中 \(S_n\) 是 \(\{a_n\}\) 的前 \(n\) 项和, 求 \(\{a_n\}\) 的通项公式.

    分析和解答

    不难计算得到 \(a_1 = S_1 = 2\). 另根据已知条件, \(S_n + 1 = \dfrac{S_n+4}{S_n - 2S_{n-1}}\), 可变形得
    \[
    S_{n-1} = \dfrac{S_n^2 - 4}{2(S_n+1)}
    \]
    直接处理数列 \(\{S_n\}\), 其不动点方程为 \(x = \dfrac{x^2-4}{2(x+1)}\), 解得 \(x = -1 \pm \sqrt3 i = 2 e^{i \cdot \frac{\pm 2\pi}3}\). 记 \(\omega_1 = e^{i \cdot \frac{2\pi}3}\), \(\omega_2 = e^{i \cdot \frac{-2\pi}3}\), 则不动点方程的两根分别为 \(2\omega_1\) 和 \(2\omega_2\).

    \(\omega_{1,2}\) 为三次单位根, 有

    1. \(\omega_1^2 + \omega_1 + 1 = \omega_2^2 + \omega_2 + 1 = 0\);
    2. \(\omega_1 = \omega_2^2\), \(\omega_2 = \omega_1^2\);
    3. \(\omega_1 = \overline{\omega_2}\)

    分别用两个不动点处理数列 \(\{S_n\}\) 的递推关系, 有
    \[
    \begin{aligned}
    S_{n-1} - 2 \omega_1 &= \dfrac{(S_n - 2 \omega_1)^2}{2(S_n+1)} \\
    S_{n-1} - 2 \omega_2 &= \dfrac{(S_n - 2 \omega_2)^2}{2(S_n+1)}
    \end{aligned}
    \]
    两式相除得
    \[
    \dfrac{S_{n-1} - 2 \omega_1}{S_{n-1} - 2 \omega_2} = \left( \dfrac{S_n - 2 \omega_1}{S_n - 2 \omega_2} \right)^2
    \]

    比较两边的模和辐角主值. 由于 \(\forall n \in \mathbb N^*, S_n \in \mathbb R\), 故 \(S_n - 2\omega_1 = \overline{S_n - 2\omega_2}\), 于是 \(\left| \dfrac{S_n - 2 \omega_1}{S_n - 2 \omega_2} \right| = 1\), 模的条件自然满足.

    设 \(S_n - 2\omega_1\) 的辐角主值为 \(\theta_n\), 则 \(S_n - 2\omega_2\) 的辐角主值为 \(-\theta_n\), 于是
    \[
    \dfrac{S_n - 2 \omega_1}{S_n - 2 \omega_2} = e^{i \cdot 2\theta_n}
    \]
    因此
    \[
    e^{i \cdot 2\theta_{n-1}} = e^{i \cdot 4\theta_n}, \quad 2\theta_{n-1} = 4\theta_n + 2k\pi
    \]
    由于 \(\{a_n\}\) 是正数数列, \(\{S_n\}\) 也是正数数列且单调递增, 故复数 \(S_n - 2\omega_1\) 的实部 \(\Re(S_n - 2\omega_1) = S_n+1\) 恒为正且单调递增, 而虚部 \(\Im (S_n - 2\omega_1) = -\sqrt3\) 为负数常值, 故 \(\{\theta_n\}\) 都是第四象限角且单调递增, 因而只能是
    \[
    2\theta_{n-1} = 4\theta_n, \quad \theta_n = \dfrac1{2^{n-1}} \cdot \theta_1
    \]
    其中, \(\theta_1\) 是 \(S_1 - 2 \omega_1 = 2 - 2 \omega_1 = 3-\sqrt3i\) 的辐角, \(\theta_1 = -\dfrac\pi6\). 于是 \(\theta_n = -\dfrac\pi{3\cdot 2^n}\).

    由 \(\dfrac{S_n - 2 \omega_1}{S_n - 2 \omega_2} = e^{i \cdot 2\theta_n}\) 反推得
    \[
    \begin{aligned}
    S_n &= \dfrac{2 (\omega_1 - \omega_2 \cdot e^{i \cdot 2\theta_n})}{1 - e^{i \cdot 2\theta_n}} \\
    &= 2\cdot \dfrac{(\omega_1 - \omega_2 \cdot e^{i \cdot 2\theta_n})(1 - e^{-i \cdot 2\theta_n})}{(1 - e^{i \cdot 2\theta_n})(1 - e^{-i \cdot 2\theta_n})} \\
    &= 2\cdot \dfrac{\omega_1 + \omega_2 - \omega_1 \cdot e^{-i \cdot 2\theta_n} - \omega_2 \cdot e^{i \cdot 2\theta_n}}{1 - e^{i \cdot 2\theta_n} - e^{-i \cdot 2\theta_n} + 1} \\
    &= 2\cdot \dfrac{ -1 -2 \cos \left( \dfrac{2\pi}3 - 2\theta_n \right)}{2 - 2\cos2\theta_n} \\
    &= \dfrac{-1+\cos2\theta_n - \sqrt3 \sin2\theta_n}{1-\cos2\theta_n} \\
    &= \dfrac{-\sqrt3 \sin2\theta_n}{1-\cos2\theta_n} -1 \\
    &= -\sqrt3 \cot \theta_n - 1 \\
    &= \sqrt3 \cot \dfrac\pi{3\cdot 2^n} - 1
    \end{aligned}
    \]
    注意到 \(S_0 = \sqrt3 \cot \dfrac\pi3 - 1 = 0\), 有 \(\forall n \in \mathbb N^*\), 都有
    \[
    \begin{aligned}
    a_n &= S_n - S_{n-1} \\
    &= \sqrt3 \left( \cot \dfrac\pi{3\cdot 2^n} - \cot \dfrac\pi{3\cdot 2^{n-1}} \right)
    = \dfrac{\sqrt3}{\sin\dfrac\pi{3\cdot2^{n-1}}}
    \end{aligned}
    \]

    注意到
    \[
    \begin{aligned}
    \cot \theta - \cot 2\theta &= \dfrac{\cos\theta}{\sin\theta} - \dfrac{\cos2\theta}{\sin2\theta} \\
    &= \dfrac{\sin2\theta\cos\theta - \cos2\theta\sin\theta}{\sin\theta \sin2\theta} \\
    &= \dfrac{\sin\theta}{\sin\theta \sin2\theta} = \dfrac1{\sin2\theta}
    \end{aligned}
    \]

    ]]>
    <![CDATA[三角不等式/极值问题]]> 2018-06-13T18:21:28+08:00 https://cshishaliu.github.io/15288852882913.html TODO: 本题确认伪证了

    已知 \(A,B,C\) 为三角形三内角. 求 \(\dfrac{\cos^2 A}{1+\cos A} + \dfrac{\cos^2 B}{1+\cos B} + \dfrac{\cos^2 C}{1+\cos C}\) 的最小值.

    Qer: zhangboxin 20180612

    解答

    设 \(t_1 = \tan \dfrac A2\), \(t_2 = \tan \dfrac B2\), \(t_3 = \tan \dfrac C2\). 则由 \(A + B + C = \pi\) 可导出
    \[
    t_1 t_2 + t_2 t_3 + t_3 t_1 = 1
    \]

    \[
    1 + t_1^2 = t_1 t_2 + t_2 t_3 + t_3 t_1 + t_1^2 = (t_1+t_2)(t_1+t_3)
    \]
    以及
    \[
    t_1^2 + t_2^2 + t_3^2 = (t_1 + t_2 + t_3)^2 - 2
    \]
    又由 \(t_1^2 + t_2^2 + t_3^2 \ge \dfrac13 (t_1 + t_2 + t_3)^2\) 知
    \[
    (t_1 + t_2 + t_3)^2 \ge 3
    \]
    当且仅当 \(t_1 = t_2 = t_3 = \dfrac{\sqrt3}3\) 时取等号.

    由万能公式
    \[
    \dfrac{\cos^2 A}{1 + \cos A} = \dfrac{(1-t_1^2)^2}{2(1+t_1^2)} = \dfrac{t_1^2 - 3}2 + \dfrac2{t_1^2 + 1} = \dfrac{t_1^2}2 + \dfrac2{(t_1+t_2)(t_1+t_3)} - \dfrac32
    \]
    从而
    \[
    \begin{aligned}
    \text{原式}
    &= \dfrac{t_1^2 + t_2^2 + t_3^2}2 + \dfrac{2[(t_2+t_3)+(t_3+t_1)+(t_1+t_2)]}{(t_1+t_2)(t_2+t_3)(t_3+t_1)} - \dfrac92 \\
    &= \dfrac{(t_1 + t_2 + t_3)^2}2 + \dfrac{4(t_1+t_2+t_3)}{(t_1+t_2)(t_2+t_3)(t_3+t_1)} - \dfrac{11}2 \\
    \end{aligned}
    \]
    由于
    \[
    \begin{aligned}
    (t_1+t_2)(t_2+t_3)(t_3+t_1)
    &\le \left[ \dfrac{(t_1+t_2)+(t_2+t_3)+(t_3+t_1)}3 \right]^3 \\
    & = \dfrac8{27} (t_1+t_2+t_3)^3
    \end{aligned}
    \]
    当且仅当 \(t_1 = t_2 = t_3\) 时取等号. 所以
    \[
    \text{原式} \ge \dfrac{(t_1 + t_2 + t_3)^2}2 + \dfrac{27}{2(t_1+t_2+t_3)^2} - \dfrac{11}2
    \]
    又由于 \((t_1 + t_2 + t_3)^2 \ge 3\), 故
    \[
    \text{原式} \ge \dfrac32 + \dfrac{27}{2\times3} - \dfrac{11}2 = \dfrac12
    \]
    当且仅当 \(t_1 = t_2 = t_3 = \dfrac{\sqrt3}3\) 时取等号, 此时 \(A = B = C = \dfrac\pi3\).

    ]]>
    <![CDATA[一组高中联赛二试模拟/选拔题 `zhangboxin|20180516`]]> 2018-05-17T16:37:21+08:00 https://cshishaliu.github.io/15265462412456.html TODO: 微博图床已挂, 本篇缺图, 需要找回

    TODO: 本篇内容也还没完成

    这是二试难度的问题 (rdfz 20180516 高联选拔题)

    Qer: zhangboxin 20180516

    题 1

    已知 \(I\) 为 \(\triangle ABC\) 的内心, \(D\) 为 \(AI\) 延长线上一点, 且有 \(DI=DB\), \(IB \parallel CD\), 在 \(BC\) 边上取点 \(E,F\), 使得 \(\angle BAF = \angle CAE = \dfrac12 \angle BAC\), 再取 \(AE\) 延长线上一点 \(M\), 使得 \(\angle AMC = \angle ABC\), \(G\) 为 \(IF\) 的中点, \(MI\), \(DG\) 延长线相交于 \(H\). 求证: \(A,H,B,D,M,C\) 六点共圆.

    解答

    \(D, M\) 在 \(\triangle ABC\) 外接圆上不难证明, 下面只证明 \(H\) 也在外接圆上.
    对 \(\triangle AFI\) 和 TODO

    题 2

    已知实数 \(a,b,c\) 满足 \(abc = -1\), 求证: \(\dfrac{a^{10}}{a^6 + 1} + \dfrac{b^{10}}{b^6 + 1} + \dfrac{c^{10}}{c^6 + 1} \ge \dfrac32\).

    解答

    我们证明这样一个不等式问题: 若实数 \(a,b,c\) 满足 \(abc = 1\), 则 \(\dfrac{a^5}{a^3 + 1} + \dfrac{b^5}{b^3 + 1} + \dfrac{c^5}{c^3 + 1} \ge \dfrac32\).

    事实上, 若 \(abc=1\), 则 \(\dfrac{a^5}{a^3+1} = \dfrac{a^5}{a^3+abc} = \dfrac{a^4}{a^2+bc}\), 而 \(\dfrac{a^4}{a^2+bc} + \dfrac{a^2+bc}4 \ge a^2\), 有 \(\dfrac{a^4}{a^2+bc} \ge \dfrac{3a^2-bc}4\), 故
    \[
    \begin{aligned}
    \sum \dfrac{a^4}{a^2+bc}
    &\ge \sum \dfrac{3a^2-bc}4 = \dfrac{a^2+b^2+c^2}2 + \dfrac{a^2+b^2+c^2 - ab - bc - ca}4 \\
    &\ge \dfrac{a^2+b^2+c^2}2 \ge \dfrac{3 \sqrt[3]{a^2b^2c^2}}2 = \dfrac32
    \end{aligned}
    \]
    其中 \(\sum\) 表示循环和, 等号成立的充分必要条件是 \(a=b=c=1\).

    题 3

    平面上任给 5 个点, 其中任意 3 点不共线, 任意 4 点不共圆. 若某三个点所组成三角形的外接圆, 使得另外两点分别在圆的内外, 则称这个三角形是“好的”, 设“好的”三角形的个数为 \(n\), 求 \(n\) 的所有可能值.

    题 4

    对于任意正整数 \(n\), 定义 \(a_n^{(i)} = \left| \left\{ t \middle| t = C_n^k, t \mod 3 = i, 0 \le k \le n \right\} \right|\), 求证: \(a_n^{(1)} > a_n^{(2)}\).

    ]]>
    <![CDATA[Poncelet 定理: 圆和抛物线 `zhangboxin|20180409`]]> 2018-04-16T13:35:34+08:00 https://cshishaliu.github.io/15238569346799.html 已知圆 \(C_1 \colon x^2+y^2 = 1\) 和抛物线 \(C_2 \colon y = x^2 - 2\). \(P,Q,R\) 是抛物线 \(C_2\) 上的三个不同的点, 且直线 \(PQ\) 和 \(PR\) 都是圆 \(C_1\) 的切线. 求证: \(QR\) 也是圆 \(C_1\) 的切线.

    Qer: zhangboxin 20180409

    解答

    首先给出如下引理 (证明放在最后面):

    对于抛物线 \(C_2 \colon y = x^2 - 2\) 上的两个不同的点 \(M(s, s^2 - 2)\) 和 \(N(t, t^2-2)\) (其中 \(s \ne t\)),
    直线 \(MN\) 与圆 \(C_1 \colon x^2+y^2 = 1\) 相切的充分必要条件是
    \[
    s^2 t^2 - s^2 - t^2 + 2st + 3 = 0
    \]

    原题证明如下:

    设 \(P,Q,R\) 三点的坐标分别为 \(P(p, p^2 - 2)\), \(Q(q, q^2 - 2)\) 和 \(R(r, r^2 - 2)\) (其中 \(p,q,r\) 两两互不相等).

    则由 \(PQ,PR\) 与圆 \(C_1\) 相切得\[
    \begin{cases}
    p^2 q^2 - p^2 - q^2 + 2pq + 3 = 0 \\
    p^2 r^2 - p^2 - r^2 + 2pr + 3 = 0
    \end{cases}
    \]
    将这两个等式变形为
    \[
    \begin{cases}
    (p^2 - 1) q^2+ 2pq - (p^2 - 3) = 0 \\
    (p^2 - 1) r^2+ 2pr - (p^2 - 3) = 0
    \end{cases}
    \]
    由于 \(q \ne r\), 故可将 \(q,r\) 看做二次方程
    \[
    (p^2 - 1) x^2+ 2px - (p^2 - 3) = 0
    \]
    的两根, 因此由韦达定理
    \[
    \begin{cases}
    q+r = -\dfrac{2p}{p^2-1} \\
    qr = -\dfrac{p^2-3}{p^2-1}
    \end{cases}
    \]
    于是
    \[
    \begin{aligned}
    &\phantom{{}={}} q^2 r^2 - q^2 - r^2 + 2qr + 3 \\
    &= (qr)^2 - (q+r)^2 + 4qr + 3 \\
    &= \left( -\dfrac{p^2-3}{p^2-1} \right)^2 - \left( -\dfrac{2p}{p^2-1} \right)^2 + 4 \left( -\dfrac{p^2-3}{p^2-1} \right) + 3 \\
    &= \cdots \\
    &= 0
    \end{aligned}
    \]
    因此根据引理知, 直线 \(QR\) 与圆 \(C_1\) 相切.

    引理的证明

    对于抛物线 \(C_2 \colon y = x^2 - 2\) 上的两个不同的点 \(M(s, s^2 - 2)\) 和 \(N(t, t^2-2)\) (其中 \(s \ne t\)), 直线 \(MN\) 的方程为1
    \[
    MN \colon (s+t)x - y - (st+2) = 0
    \]

    直线 \(MN\) 与圆 \(C_1 \colon x^2+y^2 = 1\) 相切的充分必要条件是2
    \[
    \dfrac{|st+2|}{\sqrt{(s+t)^2+1}} = 1
    \]

    这个关系式可等价的变形为
    \[
    s^2 t^2 - s^2 - t^2 + 2st + 3 = 0
    \]

    即证.


    1. 写出两点式化简, 即可得到. 

    2. 即圆心 \(O(0,0)\) 到直线 \(MN\) 的距离等于半径 \(1\). 

    ]]>
    <![CDATA[平面几何 `duyan|20180123|2`]]> 2018-01-23T14:17:57+08:00 https://cshishaliu.github.io/15166882779923.html TODO: 微博图床已挂, 本篇缺图, 需要找回

    已知 \(H\) 为 \(\triangle ABC\) 的垂心, 过 \(H\) 的直线交 \(BC, AB\) 于 \(D,Z\), 过 \(H\) 且垂直于 \(ZH\) 的另一条直线交 \(BC, AC\) 于 \(E,X\), 点 \(Y\) 使得 \(DY \parallel AC, EY \parallel AB\). 求证: \(X,Y,Z\) 三点共线.

    Qer: duyan 20180123

    解答

    延长 \(YD\) 交 \(AB\) 于 \(G\), 延长 \(YE\) 交 \(AC\) 于 \(F\). 比较 \(\triangle GYZ\) 和 \(\triangle FXY\), 已有两边对应平行, 故只需证明它们相似即可. 下面设法证明 \(\dfrac{GZ}{GY} = \dfrac{FY}{FX}\), 亦即证明 \(GZ \cdot FX = GY \cdot FY\).

    记 \(\angle HDE = \alpha\), \(\angle HED = \beta\), 有 \(\alpha + \beta = 90^\circ\). 另外, 下面直接将 \(\triangle ABC\) 的三内角简记为 \(A,B,C\).

    在 \(\triangle GDZ\) 中, 有
    \[
    \dfrac{GZ}{GD} = \dfrac{\sin \angle GDZ}{\sin \angle GZD} = \dfrac{\sin (C+\alpha)}{\sin(B-\alpha)}
    \]
    在 \(\triangle FEX\) 中, 有
    \[
    \dfrac{FX}{FE} = \dfrac{\sin \angle FEX}{\sin \angle FXE} = \dfrac{\sin (B+\beta)}{\sin(C-\beta)}
    \]

    \[
    \begin{aligned}
    \dfrac{GZ}{GD} \cdot \dfrac{FX}{FE}
    &= \dfrac{\sin (C+\alpha)}{\sin(B-\alpha)} \cdot \dfrac{\sin (B+\beta)}{\sin(C-\beta)} = \dfrac{\cos (C-\beta)}{\sin(B-\alpha)} \cdot \dfrac{\cos (B-\alpha)}{\sin(C-\beta)} \\
    &= \dfrac{\cos(B-C) + \cos(B+C-90^\circ)}{\cos(B-C) - \cos(B+C-90^\circ)} \\
    &= \dfrac{\cos(B-C) + \sin A}{\cos(B-C) - \sin A}
    \end{aligned}
    \]
    下面只需证明 \(\dfrac{GY}{GD} \cdot \dfrac{FY}{FE} = \dfrac{\cos(B-C) + \sin A}{\cos(B-C) - \sin A}\).

    显然, \(\dfrac{GY}{GD} \cdot \dfrac{FY}{FE} = \dfrac{BE}{BD} \cdot \dfrac{CE}{CE}\), 考虑 \(B,D,E,C\) 四点的交比1, 有

    \[
    \begin{aligned}
    \dfrac{ED}{BD} \cdot \dfrac{BC}{EC}
    &= \dfrac{\sin \angle EHD}{\sin \angle BHD} \cdot \dfrac{\sin \angle BHC}{\sin \angle EHC} \\
    &= \dfrac{\sin 90^\circ}{\sin (\alpha-90^\circ+C)} \cdot \dfrac{\sin (180^\circ - A)}{\sin (\beta-90^\circ+B)} \\
    &= \dfrac{\sin A}{\sin (B-\alpha) \sin (C-\beta)} \\
    &= \dfrac{2\sin A}{\cos (B-C) - \sin A}
    \end{aligned}
    \]

    因此
    \[
    \begin{aligned}
    \dfrac{BE}{BD} \cdot \dfrac{CE}{CE}
    &= \dfrac{(BD+ED) \cdot (DE+EC)}{BD \cdot EC} \\
    &= \dfrac{ED \cdot (BD+DE+EC) + BD \cdot EC}{BD \cdot EC} \\
    &= \dfrac{ED \cdot BC}{BD \cdot EC} +1 \\
    &= \dfrac{2\sin A}{\cos (B-C) - \sin A} + 1 \\
    &= \dfrac{\cos (B-C) + \sin A}{\cos (B-C) - \sin A}
    \end{aligned}
    \]
    证毕.


    1. 交比的相关结论可用正弦定理证明. 

    ]]>
    <![CDATA[USAMO19xx #5, 高斯函数, 不等式, 第二数学归纳法]]> 2018-01-23T12:35:51+08:00 https://cshishaliu.github.io/15166821516457.html 证明不等式:
    \[
    [nx] \ge \dfrac{[x]}1 + \dfrac{[2x]}2 + \dfrac{[3x]}3 + \dots +\dfrac{[nx]}n
    \]
    其中, \(n \in \mathbb N^*\), \([x]\) 表示不大于 \(x\) 的最大整数.

    src 第十届美国数学奥林匹克第五题

    Qer: zhangboxin 20171125

    解答


    \[
    A_n = \dfrac{[x]}1 + \dfrac{[2x]}2 + \dfrac{[3x]}3 + \dots +\dfrac{[nx]}n
    \]
    需证明 \([nx] \ge A_n\), 对 \(n\) 进行归纳 (第二数学归纳法).

    当 \(n=1\) 时显然成立.

    设 \(n \le k\) 时, 不等式均成立, 即
    \[
    A_1 \le [x], A_2 \le [2x], \dots, A_k \le [kx]
    \]
    由于
    \[
    \begin{aligned}
    (k+1) A_{k+1} - (k+1) A_k &= [(k+1)x] \\
    k A_k - k A_{k-1} &= [kx] \\
    \vdots\\
    2 A_2 - 2 A_1 &= [2x] \\
    A_1 &= [x]
    \end{aligned}
    \]
    全部累加起来有
    \[
    \begin{gathered}
    (k+1) A_{k+1} - (A_1 + A_2 + \dots + A_k) \le [x] + [2x] + \dots +[(k+1)x] \\
    (k+1) A_{k+1} \le (A_1 + A_2 + \dots + A_k) + [x] + [2x] + \dots +[(k+1)x]
    \end{gathered}
    \]

    \[
    \begin{gathered}
    A_1 + [kx] \le [x] + [kx] \le [(k+1)x] \\
    A_2 + [(k-1)x] \le [2x] + [(k-1)x] \le [(k+1)x] \\
    \vdots \\
    A_k + [x] \le [kx] + [x] \le [(k+1)x] \\
    \end{gathered}
    \]
    因此,
    \[
    \begin{gathered}
    (k+1) A_{k+1} \le (k+1) \cdot [(k+1)x] \\
    A_{k+1} \le [(k+1)x]
    \end{gathered}
    \]
    证毕.

    ]]>